Proving a real valued function does not exist

  • Thread starter Thread starter _N3WTON_
  • Start date Start date
  • Tags Tags
    Function
Click For Summary
The discussion centers on proving that no real-valued function 'u' exists under the condition that λ > 1/2, given the equation u(x) = 1 + λ∫[x to 1] u(y)u(y-x) dy. The attempt involved assuming such a function exists and manipulating the integral, leading to a derived equation for μ, which represents the integral of u over the interval. The quadratic equation formed from the analysis indicates that the discriminant must be non-negative for μ to exist, resulting in the condition λ ≤ 1/2. This conclusion contradicts the initial assumption of λ being greater than 1/2, thereby confirming that no such function can exist under the given conditions. The proof is deemed correct by the participants in the discussion.
_N3WTON_
Messages
350
Reaction score
3

Homework Statement


Show that if \lambda > \frac{1}{2} there does not exist a real-valued function 'u' such that for all x in the closed interval 0 <= x <= 1, u(x) = 1 + \lambda\int_{x}^{1} u(y)u(y-x)\,dy

Homework Equations

The Attempt at a Solution


I started off by assuming that such a function does in fact exist, so:
u(x) = 1 + \lambda\int_{x}^{1}u(y)u(y-x)\,dy
\lambda > \frac{1}{2}
0 <= x <= 1
\mu = \int_{0}^{1} u(x)\,dx
= \int_{0}^{1}\,dx + \int_{0}^{1}\int_{x}^{1} u(y)u(y-x) dy \hspace{1 mm} dx
Here, I changed the order of integration:
= 1 + \lambda\int_{0}^{1}\int_{0}^{y} u(y)u(y-x)dx \hspace{1 mm} dy
= 1 + \lambda\int_{0}^{1}u(y)\int_{0}^{y}u(y-x)dx \hspace{1 mm} dy
At this point I am a bit confused about where to go, so I am starting to think that perhaps I am not going in the right direction; I was hoping someone could give me some advice. Thanks :)
 
Physics news on Phys.org
Ok, I worked on this for a quite a while last night before bed and I think I have a solution, hopefully someone can confirm/deny...I'll start where I left off above, where I take Y constant:
z = y - x
dz = -dx
\mu = 1 + \lambda\int_{0}^{1}u(y)\int_{y}^{0} u(z)(-dz)\,dy
= 1 + \lambda\int_{0}^{1}u(y)\int_{0}^{y}u(z)dz\,dy
{Let} \hspace{1 mm} f(y) = \int_{0}^{y} u(z)\,dz \hspace{2 mm} {then,} \hspace{2 mm} f'(y) = u(y), \hspace{1 mm} f(0)=0, \hspace{1 mm} f(1) = \mu
\mu = 1 + \lambda\int_{0}^{1}f'(y)f(y)\,dy
= 1 + \frac{\lambda {\mu}^2}{2}
\lambda {\mu}^2 - 2\mu + 2 = 0
So now, if mu is going to exist, the discriminant of that quadratic cannot be negative:
4 - 8\lambda >= 0
\lambda <= \frac{1}{2}
-This contradicts the problem statement that \lambda > \frac{1}{2}. Hence, it is confirmed that for \lambda > \frac{1}{2} there does not exist u = 1 + \lambda\int_{x}^{1}u(y)u(y-x)\,dy.
 
Looks correct.
 
thanks :)
 
Question: A clock's minute hand has length 4 and its hour hand has length 3. What is the distance between the tips at the moment when it is increasing most rapidly?(Putnam Exam Question) Answer: Making assumption that both the hands moves at constant angular velocities, the answer is ## \sqrt{7} .## But don't you think this assumption is somewhat doubtful and wrong?

Similar threads

Replies
4
Views
2K
Replies
6
Views
2K
  • · Replies 10 ·
Replies
10
Views
2K
Replies
3
Views
1K
  • · Replies 2 ·
Replies
2
Views
2K
  • · Replies 11 ·
Replies
11
Views
3K
Replies
5
Views
2K
Replies
19
Views
2K
  • · Replies 6 ·
Replies
6
Views
2K
  • · Replies 10 ·
Replies
10
Views
2K